michael.gorshein
Thanks Received: 0
Vinny Gambini
Vinny Gambini
 
Posts: 4
Joined: March 22nd, 2012
 
 
 

Q8 - Eva: A "smart highway" system

by michael.gorshein Wed Apr 11, 2012 2:04 pm

Don't understand why the answer is C as opposed to A or even B. The only reason I can like C more than the former 2 is that it uses language consistent with the stimulus, but I would never have picked it over A before having seen the answer.
User avatar
 
ManhattanPrepLSAT1
Thanks Received: 1909
Atticus Finch
Atticus Finch
 
Posts: 2851
Joined: October 07th, 2009
 
 
 

Re: Q8 - Eva: A "smart highway" system

by ManhattanPrepLSAT1 Fri Apr 13, 2012 7:22 pm

Good question! Thanks michael.gorshein for bringing this one to the forum.

Eva claims that a "smart highway" system will lead to improved traffic flow and a decrease in the loss of money and productivity that currently result from the traffic congestion.

Necessary Assumptions often work in two different ways; linking together a chain of logic, or defending the argument from something devastating. In this case there isn't a missing link in a chain of reasoning, but there is the possibility that the "smart highway" system may not be able to accomplish what Eva suggests. Answer choice (C) defends the argument from something that would prevent improved traffic flow - and so is the correct answer!

Let's look at the incorrect answers:

(A) is not necessary. There may be breakdowns, but the "smart highway" system may lead to improved traffic flow by some other means.
(B) is too strong. It need not be true that a free flow of traffic is assured, so long as the flow of traffic is improved.
(D) is out of scope, why would "smart highway" systems need varying types of equipment?
(E) undermines the argument from the beginning. A necessary assumption will always support the conclusion - not undermine it.

Hope that helps! Let me know if that doesn't answer your question.
 
anjelica.grace
Thanks Received: 5
Jackie Chiles
Jackie Chiles
 
Posts: 41
Joined: November 17th, 2011
 
 
 

Re: Q8 - Eva: A "smart highway" system

by anjelica.grace Fri May 25, 2012 1:36 am

I got this question right but by process of elimination and still wasn't too convinced by (C).

As a general rule, if the conclusion of an argument recommends something ("should"), I try to avoid AC's that discuss the feasibility ("can/could/possible") of the action being recommended, and vice versa.

I'm realizing now, however, that approach only works for certain question types, such as inference, strengthen, weaken, but maybe not so much for necessary assumption questions.

For example, if one argues, "You should read this book," s/he must be assuming that I'm literate i.e. it is possible for me to read, is that correct?

In general, I am asking if an argument makes a recommendation, is one basic assumption that the recommendation is feasible? I usually thought that whether something COULD be done is irrelevant to whether it SHOULD be done.
 
peterbobolis
Thanks Received: 2
Vinny Gambini
Vinny Gambini
 
Posts: 9
Joined: October 09th, 2012
 
 
 

Re: Q8 - Eva: A "smart highway" system

by peterbobolis Wed Nov 28, 2012 6:46 pm

perhaps I overanalyzed this one (is that possible on LSAT?)...but it's the word "significant" in choice C that still has me convinced it's less than perfect, albeit better than the other choices.
 
schmid215
Thanks Received: 5
Jackie Chiles
Jackie Chiles
 
Posts: 36
Joined: September 03rd, 2012
 
 
 

Re: Q8 - Eva: A "smart highway" system

by schmid215 Fri Jan 25, 2013 10:25 am

peterbobolis Wrote:perhaps I overanalyzed this one (is that possible on LSAT?)...but it's the word "significant" in choice C that still has me convinced it's less than perfect, albeit better than the other choices.


I think you're right on...."significant" seems like it changes everything. All that is necessary for the argument to hold is for some improvement to be possible, because the conclusion is that we would see a decrease, not a significant decrease. It still is the best of the bunch, though. I can't believe I fell for (A).
 
MayMay
Thanks Received: 0
Forum Guests
 
Posts: 25
Joined: January 02nd, 2013
 
 
 

Re: Q8 - Eva: A "smart highway" system

by MayMay Sat Apr 06, 2013 12:30 pm

schmid215 Wrote:
peterbobolis Wrote:perhaps I overanalyzed this one (is that possible on LSAT?)...but it's the word "significant" in choice C that still has me convinced it's less than perfect, albeit better than the other choices.


I think you're right on...."significant" seems like it changes everything. All that is necessary for the argument to hold is for some improvement to be possible, because the conclusion is that we would see a decrease, not a significant decrease. It still is the best of the bunch, though. I can't believe I fell for (A).


can we get more on this?
i see that choice C is the best answer, but the "significant" is really upsetting me!
 
alex.cheng.2012
Thanks Received: 8
Forum Guests
 
Posts: 28
Joined: May 02nd, 2013
 
 
 

Re: Q8 - Eva: A "smart highway" system

by alex.cheng.2012 Sat Jun 22, 2013 9:46 pm

I feel the same about what a few of the previous posters said. I narrowed it down to B and C. I ultimately chose B because C included the word "significant."

The argument only states that it would improve traffic flow (and drivers' tempers) and decrease "considerable" loss of money and productivity resulting from traffic congestion. I felt that the argument wasn't assuming it would bring "significant" improvement. After all, it only said it would improve traffic flow, it doesn't specify the degree of improvement. Considerable stood out, but I felt that even then, it didn't warrant "significant." If the money loss was a total of $1 trillion, and the system would help decrease that loss by $1 million, that is still a considerable amount of money saved, but not significant in comparison to the $1 trillion.

After reading what you guys said (thanks all!), I now see how B isn't the correct answer, but I still can't see how C is the correct answer.

On a tangent of what mattsherman said. B - it need not be assured free flow of traffic occurs, as long as there is an improvement. Couldn't the same be said of C? C - it need not bring about significant improvement, as long as there is an improvement.
 
LSATeater
Thanks Received: 0
Vinny Gambini
Vinny Gambini
 
Posts: 13
Joined: July 22nd, 2013
 
 
 

Re: Q8 - Eva: A "smart highway" system

by LSATeater Mon Jul 22, 2013 3:35 pm

alex.cheng.2012 Wrote:I feel the same about what a few of the previous posters said. I narrowed it down to B and C. I ultimately chose B because C included the word "significant."

The argument only states that it would improve traffic flow (and drivers' tempers) and decrease "considerable" loss of money and productivity resulting from traffic congestion. I felt that the argument wasn't assuming it would bring "significant" improvement. After all, it only said it would improve traffic flow, it doesn't specify the degree of improvement. Considerable stood out, but I felt that even then, it didn't warrant "significant." If the money loss was a total of $1 trillion, and the system would help decrease that loss by $1 million, that is still a considerable amount of money saved, but not significant in comparison to the $1 trillion.

After reading what you guys said (thanks all!), I now see how B isn't the correct answer, but I still can't see how C is the correct answer.

On a tangent of what mattsherman said. B - it need not be assured free flow of traffic occurs, as long as there is an improvement. Couldn't the same be said of C? C - it need not bring about significant improvement, as long as there is an improvement.



Good points! But, and as has been noted, C is undoubtedly the best of the bunch. In fact, I think that the "significant" qualifier is what makes this question slightly more difficult than your average necessary assumption questions and shows why we categorize these questions as necessary assumptions and not MUST be trues. Whereas with must be trues we need to only operate in absolutes, we are expected (and as this shows, also required) to sometimes be slightly relaxed in our approach to assumption questions.

In regards to your point about B, you are right again that so long as we have a "freer" traffic flow with the the smart highway, we need not be assured. But that is precisely why B is wrong: "...would assure a free traffic flow" is a blatant appeal to ABSOLUTE 100% assurance which is not necessary. With C, there is no such absolute appeal because "significant improvement" is more vague about the degree of improvement which is why it does not commit the same error as choice B.

I really used to hate some of these assumption questions but thinking about the choices by inspecting for vagueness and absolutes ultimately made choice C totally palatable for me.

Hope this helped!
 
ptewarie
Thanks Received: 36
Forum Guests
 
Posts: 38
Joined: October 01st, 2012
 
 
 

Re: Q8 - Eva: A "smart highway" system

by ptewarie Fri Aug 09, 2013 2:05 pm

A few things to reiterate whenever going over Assumptions:

1. Assumptions are not strengtheners( although all strengtheners are assumptions). This is key! A lot of confusion can stem from thinking that what could strengthen argument is what is required to make the argument true. This is NOT the same thing.

ie: Poison is harmful , so Chemical X is harmful

1:
Assumption: Chemical X is a poison( this MUST be true for argument to work

2:
Strengthener: John, an A+ student in chemistry, wrote a paper arguing that Chemical is a poison

While sentence 1 strengthens the argument, it is also ABSOLUTELY necessary for the argument to be true. Sentence 2 strengthens it as well but does NOT have to be absolutely true.

When in doubt( don't rely on this always), use the denial test.
For sentence 2, John did NOT support that chemical X is a poison , does NOT make the conclusion invalid.

Like a previous post mentioned: Assumption does two things,
A. Bridge the gap from Evidence to Conclusion
B. Rules out possibility that Conclusion does not fall apart.


In this specific problem:

Evidence:
1.System gives drivers key info on traffic
2. System would(notice not saying that it WILL absolutely, but saying its likely) improve traffic flow, congestion around city and drivers tempers, loss of money and productivity

Conclusion: System should be installed.

AC:
A: Smart highways would not cause congestions.
-> Note, this is a possible strengthener. If it doesn't cause congestion then we have an additional reason for using it. HOWEVER, it is NOT necessary for argument to be true.
If it DOES cause congestion, it weakens the argument, but it does not necessarily destroy the argument that it COULD improve traffic flow.

Why? One reason is that it notes that it gives people INFO on congestions and Alternate routes. If it would claim that smart highways prevent congestions only than this would be a problem. It gives only Info AND gives people signals about alternative routes so it could STILL despite causing congestions, give people
INFO about congestions AND give people info on alternative routes.

C. if C is true, then improvement is IMPOSSIBLE ( ie: not possible to provide alt routes since traffic around cities is too confgestion, neither will info be able to prevent congestion) Since both the goals of the smart highway are destroyed if true, then this is a weakener but also the NECESSARY INFO.

Morale of this stem:
1. Watch out ALWAYS for qualifier words.
In this stem it says " we can infer that it would" and its about INFORMATION which will allow people to make good choices. It does not say it will DIRECTLY clear congestion.

2. Remember to ask yourself whether something is only strengthening it but not necessarily assuring that it is true.
REMEMBER: Assumptions are ALWAYS strengtheners but Strengtheners are not always assumption.

Good luck,
CD
User avatar
 
WaltGrace1983
Thanks Received: 207
Atticus Finch
Atticus Finch
 
Posts: 837
Joined: March 30th, 2013
 
 
trophy
Most Thanked
trophy
Most Thankful
trophy
First Responder
 

Re: Q8 - Eva: A "smart highway" system

by WaltGrace1983 Fri Jun 27, 2014 3:49 pm

anjelica.grace Wrote:if one argues, "You should read this book," s/he must be assuming that I'm literate i.e. it is possible for me to read, is that correct?

In general, I am asking if an argument makes a recommendation, is one basic assumption that the recommendation is feasible? I usually thought that whether something COULD be done is irrelevant to whether it SHOULD be done.


Could a geek chime in on this question?

I do believe that this principle is right. There seems to always be an implicit assumption that, when arguing for a certain position, that the position in question is actually possible to take. I cannot think of a situation in which this would not apply.

In addition, as for the word "significant," I remember a geek saying in another thread (can't remember which one, sorry everyone!) that "significant" is a basically useless word on the LSAT - like "some" or "many." The implicit vagueness of the word is such that it is usually to be ignored (unless, of course, the word 'significant' is defined in some way in the stimulus itself).
User avatar
 
maryadkins
Thanks Received: 641
Atticus Finch
Atticus Finch
 
Posts: 1261
Joined: March 23rd, 2011
 
 
 

Re: Q8 - Eva: A "smart highway" system

by maryadkins Mon Jun 30, 2014 10:36 am

1) I agree that usually on the LSAT, the assumption (as in, the problem the test wants you to identify) is not "should" assuming "could." But always keep an eye out for gaps because of course it could (should? jk) still come up, just like all term shifts can come up.

2) Significant doesn't have a concrete meaning on the test. It just means what it means in real life. So there would be a term shift, say, from "significant" to any word that clearly means insignificant: trivial, unremarkable, etc.